Answered step by step
Verified Expert Solution
Link Copied!

Question

1 Approved Answer

Two rms, Firm 1 and Firm 2, compete by simultaneously choosing prices. Both rms sell an identical product for which each of 100 consumers has

image text in transcribed
Two rms, Firm 1 and Firm 2, compete by simultaneously choosing prices. Both rms sell an identical product for which each of 100 consumers has a maximum willingness to pay of $40. Each consumer will buy at most 1 unit, and will buy it from whichever rm charges the lowest price. If both rms set the same price, they share the market equally. Costs are given by c, ((1,) = lqi. Because of government regulation, rms can only choose prices which are integer numbers, and they cannot price above $40. Answer the following: a) (0.25 point) If Firm 1 chooses p1 = 31, Firm 2's best response is to set what price? [3 b) (0.25 point) If Firm 2 chooses the price determined in the previous question, Firm 1's best response is to choose what price? [3 c) ('I point) If Firm 1 chooses p1 = 5, Firm 2's best response is a range of prices. What is the lowest price in this range? [3

Step by Step Solution

There are 3 Steps involved in it

Step: 1

blur-text-image

Get Instant Access to Expert-Tailored Solutions

See step-by-step solutions with expert insights and AI powered tools for academic success

Step: 2

blur-text-image

Step: 3

blur-text-image

Ace Your Homework with AI

Get the answers you need in no time with our AI-driven, step-by-step assistance

Get Started

Recommended Textbook for

Global Strategy

Authors: Mike W. Peng

5th Edition

0357512367, 978-0357512364

More Books

Students also viewed these Economics questions

Question

8. What values do you want others to associate you with?

Answered: 1 week ago